Find the inverse Laplace transform of the following: F(s)=((s−2)e^(-s))/((s^2−4s+3))

Ty Gaines

Ty Gaines

Answered question

2022-11-05

Find the inverse Laplace transform of the following:
F ( s ) = ( s 2 ) e s ( s 2 4 s + 3 )

Answer & Explanation

Lena Gomez

Lena Gomez

Beginner2022-11-06Added 14 answers

HINT:
s 2 ( s 1 ) ( s 3 ) = 1 / 2 s 3 + 1 / 2 s 1
InjegoIrrenia1mk

InjegoIrrenia1mk

Beginner2022-11-07Added 3 answers

Well, using the convolution theorem:
(1) L s 1 [ 1 e s s 2 s 2 4 s + 3 ] ( t ) = L s 1 [ 1 e s ] ( t )     L s 1 [ s 2 s 2 4 s + 3 ] ( t )
Now, we also know:
(2) s 2 s 2 4 s + 3 = 1 2 { 1 s 1 + 1 s 3 }
So, we get:
(3) L s 1 [ 1 e s ] ( t )     { 1 2 ( L s 1 [ 1 s 1 ] ( t ) + L s 1 [ 1 s 3 ] ( t ) ) }

Do you have a similar question?

Recalculate according to your conditions!

New Questions in Differential Equations

Ask your question.
Get an expert answer.

Let our experts help you. Answer in as fast as 15 minutes.

Didn't find what you were looking for?